GRE网络课堂学习笔记-逻辑[7]

文章作者 100test 发表时间 2007:02:25 20:09:04
来源 100Test.Com百考试题网


第四讲 规则题

1. Questions 12-17

A hobbyist is stocking her aquarium with exactly three fish of different types and with exactly two species of paints. The only fish under consideration are a G , an H , a J , a K and an L and the only kinds of plants under consideration are of the species W , X , Y , and Z . She will observe the following conditions:
If she 0selects the G, she can 0select neither the H nor a Y.
She cannot 0select the H unless she 0selects the K.
She cannot 0select the J unless she 0selects a W.
If she 0selects the K, she must 0select an X.

(12). Which one of the following is an acceptable 0selection of fish and plants for the aquarium?

   Fish    Plants
(A) G, H, K   W, Y
(B) G, J, K   W, X
(C) G, J, L   X, Z
(D) H, J, L   W, Z
(E) H, K, L   Y, Z
(13). If the hobbyist 0selects the H, which one of the following must also be true?

(A) She 0selects at least one W.
(B) She 0selects at least one X.
(C) She 0selects the J, but no Ys.
(D) She 0selects the K ,but no Xs.
(E) She 0selects at least one X, but no Ys.

(14). If the hobbyist 0selects both Xs and Zs, which one of the following could be the group of fish she 0selects?

(A) G, H, K
(B) G, J, K
(C) G, K, L
(D) H, J, L
(E) J, K, L

(15). The hobbyist could 0select any of the following groups of fish for the aquarium EXCEPT:

(A) G, K, L
(B) H, J, K
(C) H, J, L
(D) H, K, L
(E) J, K, L

(16). If the hobbyist 0selects a Y, which one of the following must be the group of fish she 0selects?

(A) G, H, K
(B) H, J, K
(C) H, J, L
(D) H, K, L
(E) J, K, L

(17). The hobbyist could 0select ant of the following plant combinations EXCEPT:

(A) W and X
(B) W and Y
(C) W and Z
(D) X and Y
(E) X and Z

KEYS: BBCCDB



相关文章


GRE网络课堂学习笔记-逻辑[9]
GRE最新练习题第7部分
GRE网络课堂学习笔记-逻辑[7]
GRE网络课堂学习笔记-逻辑[12]
GRE最新练习题第6部分
GRE出国考试模拟试题4
GRE网络课堂学习笔记-逻辑[6]
澳大利亚华人论坛
考好网
日本华人论坛
华人移民留学论坛
英国华人论坛